Jump to content

Absolute zero-velocity measured in Sagnac Effect


geistkiesel

Recommended Posts

Can we please not fight?

 

I want to hear what geistkeissel has to say about the sagnac effect, I do not want to see them kicked off the forum by having their IP address permanently blocked.

 

I think an apology is in order, they know better than to act like that, since their level of competency is so high.

 

This person can be asked to stop nicely and will. I know things about the future, I am good like that.

 

So no more attitude please. Swansont answered your objections quite professsionally. I am simply interested in any knowledge that you or he might possess.

 

And he has been published in the IOP so your insulting him certainly isn't furthering your career.

 

Please don't let this childish thing go on, I would like to understand the Sagnac effect, and see it as you do.

 

And I want to read his responses as well.

 

Antagonist/protagonist is good, but not to extremes.

 

Regards to all

Link to comment
Share on other sites

  • Replies 94
  • Created
  • Last Reply

Top Posters In This Topic

Top Posters In This Topic

Posted Images

The detector and the photon begin to separate at moment in time t1' date=' and then the meet up at moment in time t2, and A.G. Kelly used t` to denote that amount of time.

 

But at the moment in time at which they are a distance 2R apart, the velocity vectors point in opposite directions.[/quote']

Johnny5, I quoted this from your recent post to drive home a point. I find nothing erroneous in what you have stated. It appears thaty you are taking extraordinarily lengthy routes to solve the problem as you have indicated here.

 

Your desire to be thorough is remarkable and I encourage you to continue. You may simplify your problem solving by huge leaps by looking at the problem more from a pure physics point of view and maintaining a mathematical input that is as brief as possible and recognizing that the mathematics is simply a tool of the physicist and is not the physics..

 

It took me a long time to determine solutions to problems which I felt necessary primarily for the successful passing of examinations. Then it dawned on me that the examinations were irrelevant, but if I wanted to solve mother nature it was necessary to look at nature, to look aganin and again and after I had assimilated and underrstood what I observed the problem solving became rather trivial. The difficulty that remained as a constant is in overcoming dogmantic insistence on maintaining the prevailing view of any particular subject matter of physics. Special relativity theory is a lanfmark case of arbitrary barrier building. You will not find (m)any SRT experts, for instance, ready, willing and
able
to discuss physics with you. They want to echoe Einstein's words and to draw formulae from the shelf in solving problems. Perhaps you have run againsgt this before. Well enough said about that.

 

Returning to Sagnac effects then we ask, what is the best way to set up this problem in order that we maximize the coherent information determined from the results of experiment?

 

For instance, measure motion and times from a fixed point, say 12:00 noon, label the photons cw and ccw (clock wise and counter clock wise) and t for the distance ccw moves in reaching photon emitter/detector (PED) (rotating cw) which has moved vt. The problem is trivialized and the need for vector notation is eliminated. I say this is justified using the quoted paragraphs below. The references and quote are pared from
. Basically, the Sagnac effect is not restricted to circular or two dimensional motion.

 

 

Look at it this way for a moment. The ccw arrives at the rotated photon emitter detector (PED) after moving a distance ct. The PED has moved a distance vt and from symmetry the cw is located a distance 2vt from the PED. The cw must move a disance 2vt plus a vt' distance that accounts for the motion of the PED when the cw move through the 2vt distance.Or, ct' = 2vt + vt', ot t' = t (2v)/(c -v) or in terms of velocity, v = ct'/(2t + t').

 

Ask yourself, what time is required time T for photons to circle the ring and return to 12:00 noon (as if the PE were stationary)? There are a couple of ways to do this, the easiest is to equate distances in forms you have already construdtec. setting 2piR = D, the circumference) we see that D = ct + vt, hence t = D/(c + v). Now we have defined time where it most properly resides, that is, in units of distance and velocity. Here, however, we get a bonus . We measure the time of motion and from the distance D expression, v = (D - ct)/t. We have only to measure the time for the linear distance traveled over a distance D to determine the one way uniform velocity of the object.

 

This has some interesting consequences. The motion of the light is seen consistently constant regardless of the motion of the source. This proves that light moves independently of the source, yet relative motion of photon and frame is not uiniversally c, it is either c + v or c - v. No component of the source motion is imposed on the light motion.

 

Sagnac effects are not restricted to curved, or two dimensional motion - the effect is a linear effect. For instance, light that is directed perpendicular to a moving optically-flat mirror will reflect to the identical tracjectory of the pre-reflected light beam. Take this bit of physics and modify the Michelson-Morely mathematical models for the expected time-of-flight for the beam moving orthogonal to the direction of motion of the interferometer frame and see what you get. How many times have you seen the MM mathematics indicate that the orthogonal beam is directed in a triangular path? This is a huge error echoed by a scientific community content to accept the printed word withoutr question.

 

The quoted material:

 

"The explanation of the Sagnac effect is simple for the inertial frame of reference. The motions of the mirrors during the light transit time between mirrors causes the clockwise and counter-clockwise waves to be reflected at different points of space, which leads to an optical path difference. (6)

 

This simplistic explanation can be belied by simply considering the light path to be tangent to a reflecting cylinder-that is to replace the individual mirrors by one cylindrical mirror. In that case the cylindrical mirror need not rotate at all and Browne's "simple" explanation fails. (7) This also nullifies Browne's proof that the aether cannot rotate around the earth. (8) Ives concludes his analysis of the Sagnac effect with the statement:

 

"[if the observer's] apparatus rotates with respect to the stars he will observe a Sagnac effect, if it does not, then no matter how great a relative rotation it exhibits with respect to its material surroundings, there will be no effect." (9)

 

At first sight Ives' statement would appear to counter the geocentric position, but it does not do so for Ives states that apparatus should rotate "with respect to the stars." This indicates relative rotation and is true whether we view the stars as stationary and the apparatus as rotating or whether we view the apparatus as stationary and the stars as rotating about it.

 

Over the intervening decades since Sagnac's result was published, several variations on his original experiment have been formed. The variations have been designed in an effort to either confirm or deny Sagnac's result. Most such experimental variations have involved things like conducting the experiment with apparatus in a vacuum, or else inside some medium other than air; or to have the medium rotate while the mirrors are held stationary with respect to the earth. All of these variations agree with Sagnac's original result.(10) Still another variation was that of Dufour and Prunier who kept the light source and observer separated from and not moving with the turntable on which the mirrors were mounted.(11) Their result was the same as Sagnac's."

 

References:

 

2) Sagnac, G. 1913. Comptes Rendus, 157:708 & 1410.

 

3) Ives, H.E., 1938. Jrnl. of the Optical Soc. of Am., 28:296

 

4) Post, E.J., 1967. Rev. Mod. Phys., 39:475

 

5) Browne, P.F., 1977. Jrnl. of Phys. A: Math and Gen., 10:727.

 

6) ibid p.739

 

7) Ives, 1938. Op cit., p.297

 

8) Browne, 1977. Op cit., p. 740

 

9) Ives, 1938. Op cit., p. 299

 

10) Pagamy, B. 1928. Ann.d. Phys., 85:244

 

11) Dufour, A. & F. Prunier, 1937. Comptes Rendus, 204, 1925. Also, 1942.

 

Jrnl.Phys. et Radium, 3:153 and S. Marinov, 1978. Found Phys., 8:137.

 

Link to comment
Share on other sites

Your reply to my post suggests I have an attitude problem. [...] No' date=' you just directed your comments to me and my "attitude". [/quote']

Yes, mainly because it´s your thread here and because your posts were the only ones I read. I don´t really have to read Swansonts´posts to know he´s telling you that SR is experimentally verified quite well. And I don´t have to read his posts here to know that he´s often a bit short when it comes to explanations and a bit harsh on other peoples beloved "I can disprove [add any widely accepted theory here]" ideas but not offensive.

 

For your attitude, there´s two points in it:

- The first one is rather obvious: Flaming at other people is simply a big no-no in a forum like this. I think you know yourself that you could have used another form to express that you didn´t like Swansonts´posts.

- The 2nd one is less obvious but more important when we get back to the topic of your thread: I think you haven´t understood relativity, yet. Your posts sound like you got your knowledge of SR from cross-reading hundreds of pages in the internet (made by people who also didn´t understand relativity but wanted to make a cool website) and making up your own weird theory-mix out of it.

Now when your understanding of relativity leads to a problem (whatever this might be, I find it very hard to follow you as I allready said a few times now) there´s three possible reasons for it:

a) Your thought experiment is flawed

b) Relativity leads to problems.

c) You didn´t understand Relativity correctly.

Due to lack of understanding what you say I cannot tell which is true but you don´t make the impression as if you considered c) as a valid option. That´s also what I implied when I said you should work on your attitude.

 

To summarize it: With "you should work on your attitude" I meant both, your attitude towards other people and towards science.

 

One simple question: Do you accept Special Relativity Theory in the light of the Sagnac Implications that are described in this thread? or do you need more in order that you be able to make up your mind, as in making a decision as to what you believe?

I need a explanation done in flawless SR terms that shows me that SR leads to contradictions. All your current explanations were a mixup of classical view and SR view. When I look at the "two photons are emmited at the same point and reflected my mirrors"-scenario from the perspective of SR I can´t see any problems.

 

Or asked another way: Who do you support in the current discussion: Swansont or Geistkiesel.

Neither. If at all I support ideas here, not persons. I didn´t understand your point so far and I didn´t even bother reading Swansonts´posts, so far because I don´t really care much about epxerimental physics as long as it doesn´t concern me at work.

 

 

Thank you Atheist.

Np.

Link to comment
Share on other sites

Geistkiesel,

 

You seem to know alot about the Sagnac effect, you have made me aware of it, which in and of itself is an achievement, so I thank you for that. I am going to keep working on it, I like viewing physics from a historical persepective, something I didn't get a chance to do when studying it to pass an exam. It's refreshing to go about it this way. I don't have any intelligent questions yet, so I am not going to make up one just to ask you something, but eventually I will have one, which I will need help answering so stick around, and be nice to Dr Swanson.

 

Also.. if you don't mind me asking, are you male? If that is too personal don't answer it.

 

Kind regards

Link to comment
Share on other sites

How many times have you seen the MM mathematics indicate that the orthogonal beam is directed in a triangular path? This is a huge error echoed by a scientific community content to accept the printed word withoutr question.

 

 

You may not see it because you can do the derivation in more than one frame of reference. If you do it in the frame of the interferometer, you modify the lightspeed in both arms. If you do it in the ether frame, you modify the path lengths (and use the triangle) but the speed is c. You get the same answer. Does that still count as a conspiracy?

Link to comment
Share on other sites

 

I need a explanation done in flawless SR terms that shows me that SR leads to contradictions

 

I happen to have one of these, but geistkeisel also seems to know that there is a problem with SR, perhaps via some other argument. Possibly their argument is based upon the Sagnac effect, which is exactly what I am now investigating. I should have the answer in less than a week. Although it appears to me as though their argument, however it runs, is based upon linearizing the Sagnac effect, I plan to do it both ways.

 

Why would you need such an argument though?

 

Regards

Link to comment
Share on other sites

I happen to have one of these' date=' but geistkeisel also seems to know that there is a problem with SR, perhaps via some other argument. Possibly their argument is based upon the Sagnac effect, which is exactly what I am now investigating. I should have the answer in less than a week. Although it appears to me as though their argument, however it runs, is based upon linearizing the Sagnac effect, I plan to do it both ways.

 

Why would you need such an argument though?

 

Regards[/quote']

 

Because if you attempt to prove SR wrong, you can't do it with a corrupted version of SR. In other words, you have to use the whole theory - all that it implies. You can't use just part of it, even if you disagree with it.

 

The so-called "linearized" Sagnac effect is essentially equivalent to the Michelson interferometer.

Link to comment
Share on other sites

The so-called "linearized" Sagnac effect is essentially equivalent to the Michelson interferometer.

 

This will be useful to me.

 

I have never actually seen a michelson interferometer' date=' only the very loose discussion involving (c+v), (c-v).

 

I've seen the derivation before, but there is an error in it (the derivation).

 

At any rate, about umm 5 years ago, I read a book which I have, which went into detail about the device used by Fizeau to measure the speed of light, and somewhere else I read that the Michelson interferometer was quite similiar to the device used by Fizeau.

 

You say that the "sagnac effect" is equivalent to the "michelson interferometer." That isn't exactly what you mean, so what do you mean?

 

 

PS: I can find the book again, if necessary.

 

As I recall...

 

1) Fizeau wanted to measure the speed of light experimentally.

 

(Really the speed of light in an earth frame... since speed is a frame dependent quantity)

 

He had access to something that could spin very fast, at a known rpm.

 

I think the way the rpm was known was an ingenious one, but I don't recall the details right now.

 

There were cogs or something.

 

And mirrors were involved.

 

 

 

Now, the way things worked, light was aimed at a distant stationary mirror.

 

So it went back and forth a known distance.

 

And obviously that takes some amount of time. And measuring that amount of time was the hard part. As I recall, Fizeau made his measurement in the 18th century, i think 1751 or something. I will google it in a moment.

 

Then around the same time, someone else used his technique to also measure the speed of light, and came up with a similiar measurement.

 

 

Now, Michelson did his experiments in the late nineteenth century, I think 1889, I could be wrong about that, it might have been 1898, I'll check.

 

But the point is, the experiments were done roughly one century apart.

 

So exactly how did Michelson modify the original device used by Fizeau? if you know, and how do you relate a Michelson interferometer to the Sagnac Device, which appeared in 1913, approximately a decade or so after Michelson's work?

 

Regards

 

 

PS:

 

Here is one source dating Fizeau's experiment:

 

Fizeau's measurment of speed of light

 

And the relevant quote:

 

In 1849, Louis Fizeau performed the first experiment on the Earth to measure the speed of light. His apparatus consisted of a toothed wheel, a source of light, and an arrangement of lenses and mirrors that allowed light to move along a path, be reflected from a mirror and through the toothed wheel and back. The toothed wheel was set to rotating, and light’s passage through the teeth could be matched to the wheel’s speed. Fizeau’s calculations yielded a value of 3.15 x 108 m/s for the speed of light. (Jones, Childers 1990b:613).

 

Yep, a wheel with teeth. And the date was 1849. It was Foucalt who did it slightly later 1862, and Albert Michelson slightly later at 1880. According to this source.

 

Here are two adjacent diagrams, comparing Fizeau's technique to Foucalt's:

 

Measuring light speed

 

Oddly, the link above has the date of Fizeau's measurement as 1851, disagreeing with the 1862 value given at the other site.

 

Here is a site devoted to a timeline of electromagnetism/optics:

 

Timeline of Electromagnetism and Optics

 

So all three experimenters performed their experiments in the nineteenth century.

 

Here is a quote from the site above:

 

The first successful measurement of the speed of light using an earthbound apparatus was carried out by Quick Facts about: Hippolyte Fizeau

Quick Summary not found for this subjectHippolyte Fizeau in 1849. Fizeau directed a beam of light at a mirror several thousand metres away, and placed a rotating cog wheel in the path of the beam from the source to the mirror and back again. At a certain rate of rotation, the beam could pass through one gap in the wheel on the way out and the next gap on the way back. Knowing the distance to the mirror, the number of teeth on the wheel, and the rate of rotation, Fizeau measured the speed of light as 313,000 kilometres per second.

 

 

FIZEAU, HIPPOLYTE

Armand Hippolyte Louis Fizeau (1819-1896) was a French physicist who was the first person to measure the speed of light on the Earth's surface. He measured the speed of light in 1849 using a device that consisted of a light, a toothed wheel and a distant mirror. He calculated light's speed by adjusting the speed of the wheel (the distance between wheel and mirror was 5 mi/8 km) so that the time it took the wheel to move the width of one tooth was equal to the time it took the light to travel from the wheel to the mirror and back again. He also measured the speed of light in other media, and found that light travels faster in air than in water. Fizeau also realized that the motion of a star affects its spectrum. He also did early work in daguerreotype photography.source

 

Fizeau had control over the rotation rate, and adjusted that until...

 

... the time it took the wheel to move the width of one tooth, was equal to the time it took light from a candle to travel a distance of about 10 miles, 5 to a mirror and 5 back.

 

It's hard to imagine this experiment actually being done, one would think the intensity of the light too small.

 

It's not like he had a flashlight.

 

Here is a link which shows how Fizeau incorporated lenses into the experiment, as well as commenting on the fact that one mirror was partially transmitting, partially reflecting, and another was "fully silvered."

 

I've read that before.

 

The article above, says that the wheel had 720 teeth.

 

360+360=720

 

Hence fizeau wanted his mathematical analysis to be simple. He thought ahead. Reason being, 360 has a lot of integer factors.

 

Right now I am reading about Michelson's 1927 Mt. Wilson experiment. I saw a picture of a giant iron interferometer too.

Link to comment
Share on other sites

This will be useful to me.

 

I have never actually seen a michelson interferometer' date=' only the very loose discussion involving (c+v), (c-v).

 

I've seen the derivation before, but there is an error in it (the derivation).

 

At any rate, about umm 5 years ago, I read a book which I have, which went into detail about the device used by Fizeau to measure the speed of light, and somewhere else I read that the Michelson interferometer was quite similiar to the device used by Fizeau.

 

You say that the "sagnac effect" is equivalent to the "michelson interferometer." That isn't exactly what you mean, so what do you mean?

[/quote']

 

What's the error in the derivation?

 

I said the so-called "linearized" Sagnac effect was essentially a Michelson inteferometer. The Sagnac effect is inherently a rotational effect, so saying you want to look at it in one dimension, i.e. linearize it, results in an oxymoron.

 

If light behaves as you seem to think - that the overall speed is c+v or c-v, when the source moves at v with respect to some preferred frame (the ether), then you should end up with a fringe shift in the Michelson interferometer that is dictated by v.

Link to comment
Share on other sites

What's the error in the derivation?

 

I said the so-called "linearized" Sagnac effect was essentially a Michelson inteferometer. The Sagnac effect is inherently a rotational effect' date=' so saying you want to look at it in one dimension, i.e. linearize it, results in an oxymoron.

 

If light behaves as you seem to think - that the overall speed is c+v or c-v, when the source moves at v with respect to some preferred frame (the ether), then you should end up with a fringe shift in the Michelson interferometer that is dictated by v.[/quote']

 

I was trying to understand the derivation, I had a book, they were using S and S`, and giving a presentation. And I already had formulated a clear argument that SR self contradicts. I went and found a book, covering MM, the reason I was even looking for the book in the first place, is because armed with the logic, I figured naturally the MM derivation contains an error.

 

I read and read... and stopped reading the moment i saw the error.

 

I still have the book, but as I recall, the authors switched frames, and mixed up frame measurements somehow.

 

In other words...

 

 

They defined something like this say:

 

 

A = B/C

 

 

but B was measured in frame S

 

and C was measured in frame S`

 

the derivation contained an error, as I've said.

Link to comment
Share on other sites

I was trying to understand the derivation' date=' I had a book, they were using S and S`, and giving a presentation. And I already had formulated a clear argument that SR self contradicts. I went and found a book, covering MM, the reason I was even looking for the book in the first place, is because armed with the logic, I figured naturally the MM derivation contains an error.

 

I read and read... and stopped reading the moment i saw the error.

 

I still have the book, but as I recall, the authors switched frames, and mixed up frame measurements somehow.

 

In other words...

 

 

They defined something like this say:

 

 

A = B/C

 

 

but B was measured in frame S

 

and C was measured in frame S`

 

the derivation contained an error, as I've said.[/quote']

 

You wanted to refute SR (or confirm your refutation) by finding an error in the MM derivation? That's interesting, because the MM derivation has no dependence on SR. It can't - the experiment was done almost 20 years before SR was published. The Michelson-Morley experiment was done by a bunch of people who were firmly convinced that light speed was not isotropic and that there was a preferred frame of reference.

 

Show us the error.

Link to comment
Share on other sites

 

That´s of course not true. Events (=Points in spacetime) seperated by a spacelike distance can have any order in time if you chose a suitable frame of reference. In fact' date=' one could claim that this is the reason the whole thing is called Theory of Relativity. Btw.: Of course I think that the photons arrive at the same time in a reference frame in which L and R are at rest. Everything else wouldn´t make much sense, as far as I can see it.[/quote']

 

Atheist, here is the post I found in your first post in this thread (above).

 

 

Here is what I said you said:

I am speaking in the first line:

""No the time difference is never the same "delta". It is always simultaneous. At least that is what the loudest of the SRTist are saying.

 

[then you speak]

"I get the impression that you think this experiment disproofs relativity because relativity said the two photons must allways arrive at the same time or at least with the same time-difference.

That´s of course not true."

 

As you can see I got mixed up and included my statement in something claimed was said by yourself. I was referring to the SRT claim that the photons would arrive at the L and R clocks simultahneously which would give the illusion of "rest motion".

 

AS long as we are here can we clear up the point being discussed here?

 

No, I do not claim what you said. As I rememeber, and the only thing that makes sense, is that under the conditions of the Sagnac arrangement described in the opneing thread, under any velocity V > 0 of the frame (wrt the embankment say), the photons will never be meaured the same instant at L and R, either by stationary frame clocks, obviously, or by clocks on the moving frame. The photons will, however always arrive back at the
physical Midpoint
of the L and R emitters
simultanseously
after the the frame has moved a distance 2vt + vt'. The t' happns to be the round trip photon time difference wrt the stationary conditions, t' = 0, and the moving frame condition, t' > 0. This places the
linear
Sagnac Effect as an effective motion indicator, as opposed to an accelerometer which it is the resulting
two dimensional model
.

 

Hey, Atheist, look at this last statement this erstwhile "error" mess got us?.

 

I am going to put this one down as resulting from Atheist's instintictive probing attributes and insistance on perfect "quoting" protocols. ANd who was it that sneered at intuition?

 

After both photons have moved a distance ct, ct' is the distance the right photon must cross to arrive at R, plus the vt' due to the frame motion in this short journey, or ct' = 2vt + vt'. Remember, the lp has just arrived at L after moving ct.

 

FYI t' = t(2v)/(c - v). or in terms of velocity v = ct'/(2t + t'). t' is the time difference also in the measured arrival times of the photons at L and R.

 

 

 

Did I read you correctly that SRT does not demand the photons arrive simultaneously at L and R in the moving frame? I have had bloody knock down drag outs over this pointy, when my opponent took the view that the photons woulf always arrive simutaleously at L and R in the moving frame..

 

And if i didn''t make my self clear, I undetrstand SRT to demand that the photons arrive simultaneously at L and R, otherwise the SRT would not justify the claim the frame was at restr wrt the staqtionaray frame, correct? This was brought to mind by your unambiguous agreement that the photons arrive simultaneosuly at L and R in the stationary frame.

 

Also, and I will be getting to this sooner or later, by SRT demanding the photons arrive simultaneously at L and R in the moving frame, that SRT effectivekly negates the concept of motion, here particularly the moving frame, of course, and the statioanry frame as it is "really at rest"! wrt the moving and once accelerated frame. The moving frame is the only frame subjected to relative motion producing accelerations. - this is an anlogue of the "twin paradox" resolution regarding the lack of reciprocity of the "equivalence of inertial frames of reference.".

 

 

.

 

I humbly apologize to you Atheist, for the quote I erroneously ascribed to you.

 

Geistkiesel
Link to comment
Share on other sites

You wanted to refute SR (or confirm your refutation) by finding an error in the MM derivation? That's interesting' date=' because the MM derivation has no dependence on SR. It can't - the experiment was done almost 20 years before SR was published. The Michelson-Morley experiment was done by a bunch of people who were firmly convinced that light speed was not isotropic and that there was a preferred frame of reference.

 

Show us the error.[/quote']

 

If and when Johnny5 shows his detected frrame swapping error in MM, I will give you my detected error. Virtually all the literature descibes the leg orthogonal to the frame motion is reflected sin the triangular trajectory where the frame "carries the beam along"as this is often described.

 

The postulates of the light are clearly stated that the photon moves isotropically regardless of the motion of the sources of the light. Secondly, to use the "go along with" interpretation would be to impose a moving frame's momentum compoinent on the photon thereby increasing the velocity of the light to c' > c. If the mirrors are properly adjusted for accurate 180 degrees relectioopn the photon would would return on the same physical trajectory path of the downward directed beam. The photons of both legs would then approach the eyepiece parallel wrt each other.

If the earh is moving at 30 km/sec (as assumed) then in 32 meters of roundt rip time of flight, the orthogonal beam would move a distance as calualted by:

32/3x10
8
= 10.6 x10
-8
seconds. This multiplied by the earth velocity of 30 km/sec returns a distance offset of .00318 meters = 3.18 mm.

 

MM should have been looking for 3.18 mm offset from the parallel (wrt the direction of motion) beam when the beams arrived at the eyepiece. This would be some sort of maximum offset. I betcha that MM & M adjuested the orthogonal mirror in oder to calaibrate the interferometer to a null positionj. After all they were using the wrong assumed physics for the reflection of light from an orthogonallyl moving mirror.

Link to comment
Share on other sites

If and when Johnny5 shows his detected frrame swapping error in MM' date=' I will give you my detected error. Virtually all the literature descibes the leg orthogonal to the frame motion is reflected sin the triangular trajectory where the frame "carries the beam along"as this is often described.

 

The postulates of the light are clearly stated that the photon moves isotropically regardless of the motion of the sources of the light. Secondly, to use the "go along with" interpretation would be to impose a moving frame's momentum compoinent on the photon thereby increasing the velocity of the light to c' > c. If the mirrors are properly adjusted for accurate 180 degrees relectioopn the photon would would return on the same physical trajectory path of the downward directed beam. The photons of both legs would then approach the eyepiece parallel wrt each other.

If the earh is moving at 30 km/sec (as assumed) then in 32 meters of roundt rip time of flight, the orthogonal beam would move a distance as calualted by:

32/3x10[sup']8
[/sup]
= 10.6 x10
-8
seconds. This multiplied by the earth velocity of 30 km/sec returns a distance offset of .00318 meters = 3.18 mm.

 

MM should have been looking for 3.18 mm offset from the parallel (wrt the direction of motion) beam when the beams arrived at the eyepiece. This would be some sort of maximum offset. I betcha that MM & M adjuested the orthogonal mirror in oder to calaibrate the interferometer to a null positionj. After all they were using the wrong assumed physics for the reflection of light from an orthogonallyl moving mirror.

 

 

OK, we can do a version of this.

 

We align a laser, with a spot size of a few microns into an aperture of similar size (a single-mode optical fiber) - the beam needs to be aligned to better than a micron, and deviation of a micron will cause a very noticible drop in power.

We do this at a time of day when the earth's rotational motion is aligned with the 30 km/s motion, so the earth doesn't add a perpendicular component to the speed. (i.e. if the alignment is east-west, we do this at noon). The rotational speed is about 460 m/s at the equator, and we do this a little further north, near 45 degrees, where the speed is around 333 m/s. Make the laser path length 2m, so the transit time is 6.67 ns. After 6 hours, the speed vector will have changed by 90 degrees and will add this 333 m/s vector component perpendicular to the beam, making it deflect by 2.2 microns. 12 hours after this, it will have deflected 2.2 microns in the other direction from the original spot, or a total of 4.4 microns maximum deflection. There's no way to keep the beam aligned into the optical fiber, according to your scenario.

 

Well, this experiment and ones like it are done routinely. In my lab we do this (except we don't bother to align it at a particular time of day that is most forgiving). And, not surprisingly to most, there is no deflection of the beam. Keeping it aligned all day long is simply not a problem.

 

So your theory makes a prediction which is not borne out by experiment. Time to discard the theory.

Link to comment
Share on other sites

What's the error in the derivation?

 

I said the
so-called "linearized"
Sagnac effect was essentially a Michelson inteferometer. The Sagnac effect is inherently a rotational effect' date=' so saying you want to look at it in one dimension, i.e. linearize it, results in an oxymoron.

 

If light behaves as you seem to think - that the overall speed is c+v or c-v, when the source moves at v with respect to some preferred frame (the ether), then you should end up with a fringe shift in the Michelson interferometer that is dictated by v.[/quote']

Your statenment that the Sagnac aeffect is is inherently a rotaional effect iindicates a fundamental miisundersatnding indicated by your post. The post is not correct, not by a long shot: Oxymoron? read on Swansont.

 

 

 

He is actually discussing relative motion of frame and photon wher thje relative velocity is C + v or c - v.The Sagnac effect shows that light signals emitted upon a rotating disc do not travel at the same speed with and against the direction of rotation of the disc. It has been long debated whether this same effect applies in the case were light signals are emitted upon a body in uniform translational motion. This paper shows that the Sagnac effect also applies in the latter case

 

Hatch describing that liner motion does not impose any changes in the Sagnac Effect wrt two dimensional motion.:

 

In the GPS system a non-rotating earth-centered isotropic-light-speed frame is assumed. Again, the motion of the receiver during the time the signal transits from the satellite to the receiver must be accounted for to obtain precise navigation results. In the GPS context, this effect is referred to as the one-way Sagnac effect and is blamed upon the rotation of the earth. But the receiver must account for its motion during the transit time no matter the source of the motion. It does not matter whether or not it follows a circular trajectory. The critical range which must be determined is the position of the satellite at the time the signal was transmitted and the position of the receiver at the time of its receipt. The path the receiver followed during the time of flight of the signal is completely irrelevant. This is consistent with the argument of Ives [3] that even the original Sagnac experimental results were not specifically due to rotation. Ives suggested an experimental proof designed to show the effect did not require rotation. In a beautiful modification of Ives suggestion, Ruyong Wang [4] has constructed what he calls a Fiber Optic Conveyer (FOC) which directly verifies that linear motion does not affect the speed of light.

 

 

 

.

 

When it is pointed out that those quantities do not refer to the speed of light, but rather to the sum and difference of the speed of light and the speed of some other object, both with respect to a single inertial coordinate system, which can be as great as 2c according to special relativity, the anti-scientific crackpots are undaunted, and merely proceed to construct progressively more convoluted and specious "objections". For example, they argue that each point on the perimeter of a rotating circular Sagnac device is always instantaneously at rest in some inertial coordinate system, and according to special relativity the speed of light is precisely c in all directions with respect to any inertial system of coordinates, so (they argue) the speed of light must be isotropic at every point around the entire circumference of the loop, and hence the light pulses must take an equal amount of time to traverse the loop in either direction. Needless to say, this "reasoning" is invalid, because the pulses of light are never (let alone always) at the same point in the loop at the same time during their respective trips around the loop in opposite directions.

 

 

something like the above which is more completely described in this link

 

Abstract

 

The Sagnac effect shows that light signals emitted upon a rotating disc do not travel at the same speed with and against the direction of rotation of the disc. It has been long debated whether this same effect applies in the case were light signals are emitted upon a body in uniform translational motion. This paper shows that the Sagnac effect also applies in the latter case.

This describes some contradictory claims of SRT where in one direction the frame contrracts, but motion in the oppsite direction the frame nust expand!.

 

 

 

 

 

Or try google on "sagnac effect" for a plethora of Saqnac effect papers and discussion.
Link to comment
Share on other sites

I need a explanation done in flawless SR terms that shows me that SR leads to contradictions. All your current explanations were a mixup of classical view and SR view. When I look at the "two photons are emmited at the same point and reflected my mirrors"-scenario from the perspective of SR I can´t see any problems.

 

I cannot speculate on what you did' date=' re SRT. Can you elaborate so I am able to make a coherent and rational reply? I am especially intersted in the "moving observer's point of view".

Thanx

[/indent']
Link to comment
Share on other sites

OK' date=' we can do a version of this.

 

We align a laser, with a spot size of a few microns into an aperture of similar size (a single-mode optical fiber) - the beam needs to be aligned to better than a micron, and deviation of a micron will cause a very noticeable drop in power.

We do this at a time of day when the earth's rotational motion is aligned with the 30 km/s motion, so the earth doesn't add a perpendicular component to the speed. (i.e. if the alignment is east-west, we do this at noon). The rotational speed is about 460 m/sec at the equator, and we do this a little further north, near 45 degrees, where the speed is around 333 m/sec. Make the laser path length 2m, so the transit time is 6.67 ns. After 6 hours, the speed vector will have changed by 90 degrees and will add this 333 m/sec vector component perpendicular to the beam, making it deflect by 2.2 microns. 12 hours after this, it will have deflected 2.2 microns in the other direction from the original spot, or a total of 4.4 microns maximum deflection. There's no way to keep the beam aligned into the optical fiber, according to your scenario.

 

Well, this experiment and ones like it are done routinely. In my lab we do this (except we don't bother to align it at a particular time of day that is most forgiving). And, not surprisingly to most, there is no deflection of the beam. Keeping it aligned all day long is simply not a problem.

 

So your theory makes a prediction which is not borne out by experiment. Time to discard the theory.[/quote']

Your lasers are aligned in the east-west direction I take it. You are trying to emulate the Sagnac effect by watching the beam diverge from the initial target spot? Is this correct?

If you could provide a simple sketch I would be able to follow you more coherently.

 

AT this point though I do not see any reason for the beam to diverge and point off its original direction. The Sagnac effect will measure the E/W differences in earth rotation wrt to the east to west and west to east beams of light. That is the c + v and c- v relative speed, but I see no beam divergence at this point.

 

Here is a link saying the rotational motion is significant in the Sagnac effect but earth bound experiments are not affected by orbital motion of the earth,

 

I cannot rationally respond further until I see the information I need to fully understand the arrangement at your lab. A sketch wouild certainly help.

 

Link to comment
Share on other sites

 

Johnny5, here is a good link to Dayton Miller who performed mopre than 200,000 MM experiments. Also included is an excellent and unbiased analysios of Miller's critic's especially Shanklandland that made an attempt to skuttle Miller's work.

Millers 1933 paper is invaluabel. Mine is dogged eared at this time. I keep reading it and finding things hidden.

So you want to play the game of physics? See out it is done behind closed doors.

 

 

This will cut down onn unnecessary speculations re MM, and Sagnac Effects.

 

BTW, I do not see the direct Sagnac relationship re MM and Saganc that Swnsont mentioned. maybe I am splitting hairs, but Sagnac linearized is a detector for motion, The two dimensional rotational arrangement is an effective accelerometer, gyroscope. Some functional differences anyway.

Link to comment
Share on other sites

BTW' date=' I do not see the direct Sagnac relationship re MM and Saganc that Swnsont mentioned. maybe I am splitting hairs, but Sagnac linearized is a detector for motion, The two dimensional rotational arrangement is an effective accelerometer, gyroscope. Some functional differences anyway.

[/quote']

 

One of the implications of SR is that the Michelson interferometer isn't a motion detector.

Link to comment
Share on other sites

Your lasers are aligned in the east-west direction I take it. You are trying to emulate the Sagnac effect by watching the beam diverge from the initial target spot? Is this correct?

If you could provide a simple sketch I would be able to follow you more coherently.

 

AT this point though I do not see any reason for the beam to diverge and point off its original direction. The Sagnac effect will measure the E/W differences in earth rotation wrt to the east to west and west to east beams of light. That is the c + v and c- v relative speed' date=' but I see no beam divergence at this point.

 

Here is a link saying the rotational motion is significant in the Sagnac effect but earth bound experiments are not affected by orbital motion of the earth,

 

I cannot rationally respond further until I see the information I need to fully understand the arrangement at your lab. A sketch wouild certainly help.

 

 

You're right, drawing a picture helped. The system would measure the 30 km/s speed.

 

If the system is aligned at noon or midnight, the earth's orbital velocity is in the plane of the laser. 6 hours later, it is perpendicular. The beam should deflect up or down. For the 2m path, that should be a 0.2 mm deflection.

 

Basically you'reletting the earth rotate the table, instead of doing it manually.

alignment.jpg

Link to comment
Share on other sites

I cannot speculate on what you did' date=' re SRT [so that there´s no problem in SR']. Can you elaborate so I am able to make a coherent and rational reply? I am especially intersted in the "moving observer's point of view".

Thanx

The two photons are emmited at point A, hit the mirrors at point L and R, respectively, and meet again on point B. Different coordinate systems give different coordinates for those points but the points remain the same.

Link to comment
Share on other sites

 

Your statenment that the Sagnac aeffect is is inherently a rotaional effect iindicates a fundamental miisundersatnding indicated by your post. The post is not correct' date=' not by a long shot: Oxymoron? read on Swansont.

 

 

 

He is actually discussing relative motion of frame and photon wher thje relative velocity is C + v or c - v.The Sagnac effect shows that light signals emitted upon a
rotating disc
do not travel at the same speed with and against the direction of rotation of the disc. It has been long debated whether this same effect applies in the case were light signals are emitted upon a body in uniform translational motion. This paper shows that the Sagnac effect also applies in the latter case

 

Hatch describing that liner motion does not impose any changes in the Sagnac Effect wrt two dimensional motion.:

 

In the GPS system a non-rotating earth-centered isotropic-light-speed frame is assumed. Again, the motion of the receiver during the time the signal transits from the satellite to the receiver must be accounted for to obtain precise navigation results. In the GPS context, this effect is referred to as the one-way Sagnac effect and is blamed upon the
rotation of the earth
. But the receiver must account for its motion during the transit time no matter the source of the motion. It does not matter whether or not it follows a circular trajectory. The critical range which must be determined is the position of the satellite at the time the signal was transmitted and the position of the receiver at the time of its receipt. The path the receiver followed during the time of flight of the signal is completely irrelevant. This is consistent with the argument of Ives [3] that even the original Sagnac experimental results were not specifically due to rotation. Ives suggested an experimental proof designed to show the effect did not require rotation. In a beautiful modification of Ives suggestion, Ruyong Wang [4] has constructed what he calls a Fiber Optic Conveyer (FOC) which directly verifies that linear motion does not affect the speed of light.

 

 

 

.

 

When it is pointed out that those quantities do not refer to the speed of light, but rather to the sum and difference of the speed of light and the speed of some other object, both with respect to a single inertial coordinate system, which can be as great as 2c according to special relativity, the anti-scientific crackpots are undaunted, and merely proceed to construct progressively more convoluted and specious "objections". For example, they argue that each point on the perimeter of a
rotating
circular Sagnac device is always instantaneously at rest in some inertial coordinate system, and according to special relativity the speed of light is precisely c in all directions with respect to any inertial system of coordinates, so (they argue) the speed of light must be isotropic at every point around the entire circumference of the loop, and hence the light pulses must take an equal amount of time to traverse the loop in either direction. Needless to say, this "reasoning" is invalid, because the pulses of light are never (let alone always) at the same point in the loop at the same time during their respective trips around the loop in opposite directions.

 

 

something like the above which is more completely described in this link

 

Abstract

 

The Sagnac effect shows that light signals emitted upon a
rotating disc
do not travel at the same speed with and against the direction of rotation of the disc. It has been long debated whether this same effect applies in the case were light signals are emitted upon a body in uniform translational motion. This paper shows that the Sagnac effect also applies in the latter case.

This describes some contradictory claims of SRT where in one direction the frame contrracts, but motion in the oppsite direction the frame nust expand!.

 

 

 

 

 

Or try google on "sagnac effect" for a plethora of Saqnac effect papers and discussion.

 

I don't see how you expect to show that the Sagnac effect is not due to rotation, when you can't go through an explanation of the effect without tripping over references to rotation.

 

Further, I find it interesting that you cited an explanation that seems to contradict your claim that a linear system showns a nonisotropic speed of light: Ruyong Wang [4] has constructed what he calls a Fiber Optic Conveyer (FOC) which directly verifies that linear motion does not affect the speed of light

Link to comment
Share on other sites

One of the implications of SR is that the Michelson interferometer isn't[/i'] a motion detector.

But only because the motion was "not detected"., not for any stated theoretical reasons, as far as I am able to determine (AFAIAATD). If there were any inherent systemdatici erriors in MM then the issue remains open.

And wouldn't you just predict it, I have found such an error.

Link to comment
Share on other sites

The two photons are emmited at point A, hit the mirrors at point L and R, respectively, and meet again on point B. Different coordinate systems give different coordinates for those points but the points remain the same.

I hear what you are saying, but I am not quite following the maning of it all. As you have just stated it appears applicabnle to any situation, Atheist, I believe we have an understanding. I would say the same and have in different words: Photons emitted at point A strike parallel vertical mirrors at B and C and are reflected back to point A. Of course we can take this much farther and conclude with the Saganc effect as illyustrated in the opening post.

 

Let us assume the Saganc arrangement in this thread is but a tremporary flash and that there is some constructive uise fore ther dynamics of the whole process that occured only one time, thiat is one emission, one reflection. The process begins, is emplementyed and is concluded and we all go on to the next process or event, the Saganc effect just completed remains but a vague memjory so unremarkable was its affect, This doesn't lessen the propriety of the effect and the affect on existing theories, such as SRT. AS far as I know there is no physical law requiring eternal existence for all inertial frames as a finfdamental attribute of the physical accuracy or application of the frame. In other word a frame as copnstructed as inefficient and dificult to arrange, doe s not negate the status of the frame and thagt it be relageated tot he nearest theoretical trash bin.

.

I am still cautious on your meaning here as if I have missed something you are trying to say. The fact that the same points on A , B and C can be described by an infnite number of points and coordinate systems does not obscure the conlusions of tthe Saganc effect. If this were true no coordinant system would be safe. All someone would have to do is claim "an infinite number of coordinate points and frames also describe what one has described in formal scientific logic, and that this would effectively nullify the system, But we know such infinity rich states do not give dancing lesson to the host of angels dancing on the head. of a pin.

 

I read a treatise discussing scioentific analysis of the origin of religion and the effect of the scrutiny. One point hammered home consitently the scientific community is able to describe the origins of religion
as if that explanation alone would be sufficent to bring about the downfall of religion.
If I explain SRT will SRT crumble?

 

If we look at the matter from a sheer "utility " concern, the coordinate system that works apporpriately need never be repalced until a model comes along with such a significant benefit increase that the change becomes mandatory and crucial, absolutely mecessary.

 

The Ptolemy system of prediting the positions of stellar objects did not collapse one week aftyer Gallileo and Kepler publshed their famous documents. Ptolemy lasted approximately 150 years, plus, the reason for the survival after the grand awakening given the world by the K and G was that the Ptolemy system WORKED.

 

Soemhow I think I overkilled this response, What do you think?

Link to comment
Share on other sites

I don't see how you expect to show that the Sagnac effect is not due to rotation' date=' when you can't go through an explanation of the effect without tripping over references to rotation.

 

Further, I find it interesting that you cited an explanation that seems to contradict your claim that a linear system showns a nonisotropic speed of light: [i']Ruyong Wang [4] has constructed what he calls a Fiber Optic Conveyer (FOC) which directly verifies that linear motion does not affect the speed of light[/i]

 

But look very closely what is being describeac as nonisotropic, SRT states that the relative velocity of frame and photon is always measured as C, correct" The nonisoptropic light motion is merely the recognition that the relative velocity of frame and photon is v - c or v + c. The speed of light wrt some Vf = 0 is still c. Nothing has really changed except the rhetoric. But no one is suggesting that light actually slows down or speeds up for the mere convenience of scientists constructing classical physical models of light motion. But that would be so cool if it did.
:cool:

 

Swansot. histroically the Saganc effect came from experiments on rotaion. Only through time did the application generalize to not just rotation of circular wheels, but any turning shape was seen to give the same result. But while all this weas going on the mentality was thinking in terms that the system was a rotationall system, until even Einstein suggested that the Saganc unwrap the trajectories of the turning wheel and lineralize the effect.

So see the obedient result of that command by AE in the opening post of this thread.

 

I have concluded that the rotational Saganc effect is basically an accelerometer, the linear effect a motion indicattor, meaning uniform motion and perhaps measuring accelerating frame as well..

Link to comment
Share on other sites

Create an account or sign in to comment

You need to be a member in order to leave a comment

Create an account

Sign up for a new account in our community. It's easy!

Register a new account

Sign in

Already have an account? Sign in here.

Sign In Now

×
×
  • Create New...

Important Information

We have placed cookies on your device to help make this website better. You can adjust your cookie settings, otherwise we'll assume you're okay to continue.